search
Search
Login
Unlock 100+ guides
menu
menu
web
search toc
close
Comments
Log in or sign up
Cancel
Post
account_circle
Profile
exit_to_app
Sign out
What does this mean?
Why is this true?
Give me some examples!
search
keyboard_voice
close
Searching Tips
Search for a recipe:
"Creating a table in MySQL"
Search for an API documentation: "@append"
Search for code: "!dataframe"
Apply a tag filter: "#python"
Useful Shortcuts
/ to open search panel
Esc to close search panel
to navigate between search results
d to clear all current filters
Enter to expand content preview
icon_star
Doc Search
icon_star
Code Search Beta
SORRY NOTHING FOUND!
mic
Start speaking...
Voice search is only supported in Safari and Chrome.
Navigate to
check_circle
Mark as learned
thumb_up
0
thumb_down
0
chat_bubble_outline
0
Comment
auto_stories Bi-column layout
settings

Comprehensive Guide on on Dot Product in Linear Algebra

schedule Jan 3, 2024
Last updated
local_offer
Linear Algebra
Tags
mode_heat
Master the mathematics behind data science with 100+ top-tier guides
Start your free 7-days trial now!

What is dot product?

When we are dealing with real numbers, there is only one interpretation of product - multiplication. However, in the world of vectors, there are two types of products - dot product and cross product. The dot product appears everywhere in machine learning and is therefore a must-know topic. In this guide, we will explore the dot product in depth!

Definition.

Dot product

Suppose we have the following two vectors $\boldsymbol{v}$ and $\boldsymbol{w}$:

$$\boldsymbol{v}=\begin{pmatrix} v_1\\ v_2\\ v_3\\ \end{pmatrix},\;\;\;\; \boldsymbol{w}=\begin{pmatrix} w_1\\ w_2\\ w_3\\ \end{pmatrix}$$

The dot product is defined as:

$$\boldsymbol{v}\cdot{\boldsymbol{w}}= v_1w_1+ v_2w_2+ v_3w_3$$

Note that $\boldsymbol{v}$ and $\boldsymbol{w}$ are in $\mathbb{R}^3$, but the dot product returns a scalar. The dot product is also sometimes referred to as the scalar product or the inner product.

Example.

Computing the dot product between two vectors

Compute the dot product $\boldsymbol{v}\cdot{\boldsymbol{w}}$ where $\boldsymbol{v}$ and $\boldsymbol{w}$ are defined as:

$$\boldsymbol{v}= \begin{pmatrix} 1\\2 \end{pmatrix},\;\;\;\; \boldsymbol{w}= \begin{pmatrix} 3\\4 \end{pmatrix}$$

Solution. The dot product $\boldsymbol{v}\cdot\boldsymbol{w}$ is:

$$\begin{align*} \boldsymbol{v}\cdot{\boldsymbol{w}}&= (1)(3)+(2)(4)\\ &=11 \end{align*}$$
Theorem.

Using dot product to compute the angle between two vectors

Suppose we have the following two vectors $\boldsymbol{v}$ and $\boldsymbol{w}$:

Let the angle between $\boldsymbol{v}$ and $\boldsymbol{w}$ be $\theta$. The dot product of $\boldsymbol{v}$ and $\boldsymbol{w}$ is:

$$\boldsymbol{v}\cdot{\boldsymbol{w}}= \Vert{\boldsymbol{v}}\Vert\Vert{\boldsymbol{w}}\Vert \cos{(\theta)}$$

Here, $ \Vert{\boldsymbol{v}}\Vert$ is the magnitude of $\boldsymbol{v}$. We can also rearrange this to get:

$$\cos{(\theta)}= \frac{\boldsymbol{v}\cdot{\boldsymbol{w}}} {\Vert{\boldsymbol{v}}\Vert\Vert{\boldsymbol{w}}\Vert}$$

Proof. Suppose we have the following:

Here, the vector $\boldsymbol{w}-\boldsymbol{v}$ points from $A$ to $B$ because of the geometric interpretation of vector additionlink.

The lengths of the sides are:

Using the cosine rule from elementary trigonometry, we have that:

$$\begin{equation}\label{eq:xF6GwOYW3PvcsraNGkP} \Vert{\boldsymbol{w}-\boldsymbol{v}}\Vert^2 =\Vert\boldsymbol{v}\Vert^2+ \Vert\boldsymbol{w}\Vert^2-2\Vert{\boldsymbol{v}}\Vert \Vert{\boldsymbol{w}}\Vert\cos(\theta) \end{equation}$$

Let's assume $\boldsymbol{w}$ and $\boldsymbol{v}$ are both in $\mathbb{R}^3$, though the proof can easily be generalized to $\mathbb{R}^n$. The vector $\boldsymbol{w}-\boldsymbol{v}$ is:

$$\boldsymbol{w}-\boldsymbol{v} =\begin{pmatrix} w_1-v_1\\ w_2-v_2\\ w_3-v_3\\ \end{pmatrix}$$

The square of the magnitude of $\boldsymbol{w}-\boldsymbol{v}$ is:

$$\begin{align*} \Vert{\boldsymbol{w}-\boldsymbol{v}}\Vert^2 &= \Big(\;\sqrt{(w_1-v_1)^2+ (w_2-v_2)^2+ (w_3-v_3)^2}\;\Big)^2\\ &= (w_1-v_1)^2+ (w_2-v_2)^2+ (w_3-v_3)^2 \end{align*}$$

Similarly, $\Vert{\boldsymbol{v}}\Vert^2$ and $\Vert{\boldsymbol{w}}\Vert^2$ are:

$$\begin{align*} \Vert{\boldsymbol{v}}\Vert^2 &= v_1^2+v^2_2+v^3_3\\ \Vert{\boldsymbol{w}}\Vert^2 &= w_1^2+w^2_2+w^3_3 \end{align*}$$

Substituting these squared magnitudes into \eqref{eq:xF6GwOYW3PvcsraNGkP} gives:

$$\begin{align*} (w_1-v_1)^2+ (w_2-v_2)^2+ (w_3-v_3)^2&= v^2_1+v^2_2+v^2_3+ w^2_1+w^2_2+w^2_3 -2 \Vert{\boldsymbol{v}}\Vert\Vert{\boldsymbol{w}}\Vert\cos{(\theta)} \\ v_1w_1+v_2w_2+v_3w_3&=\Vert{\boldsymbol{v}}\Vert\Vert{\boldsymbol{w}}\Vert\cos{(\theta)}\\ \boldsymbol{v}\cdot{\boldsymbol{w}}&=\Vert{\boldsymbol{v}}\Vert\Vert{\boldsymbol{w}}\Vert\cos{(\theta)} \end{align*}$$

This completes the proof.

Example.

Computing the angle between two vectors

Compute the angle between the following two vectors:

$$\boldsymbol{v}= \begin{pmatrix} 2\\ 3 \end{pmatrix},\;\;\;\; \boldsymbol{w}= \begin{pmatrix} 4\\ 1 \end{pmatrix}$$

Solution. We know from theoremlink that:

$$\boldsymbol{v}\cdot{\boldsymbol{w}}= \Vert{\boldsymbol{v}}\Vert\Vert{\boldsymbol{w}}\Vert \cos{(\theta)}$$

Rearranging the above to make $\theta$ the subject:

$$\begin{align*} \theta&= \arccos{\Big(\frac{\boldsymbol{v}\cdot{\boldsymbol{w}}} {\Vert{\boldsymbol{v}}\Vert\Vert{\boldsymbol{w}}\Vert}\Big)}\\ &=\mathrm{arccos}{\Big(\frac{(2)(4)+(3)(1)} {(2^2+3^2)^{1/2}(4^2+1^2)^{1/2}}\Big)}\\ &\approx42.5 \end{align*}$$

Therefore, the angle between vectors $\boldsymbol{v}$ and $\boldsymbol{w}$ is around $42.5$ degrees. This is confirmed visually below:

Properties of dot products

Theorem.

Dot product of two perpendicular vectors

If $\boldsymbol{v}$ and $\boldsymbol{w}$ are vectors that are perpendicular to each other, then:

$$\boldsymbol{v}\cdot{\boldsymbol{w}}=0$$

Proof. If $\boldsymbol{v}$ and $\boldsymbol{w}$ are perpendicular, then we have the following scenario:

From theoremlink, we have that:

$$\begin{align*} \boldsymbol{v}\cdot{\boldsymbol{w}}&= \Vert{\boldsymbol{v}}\Vert\Vert{\boldsymbol{w}}\Vert \cos{(\theta)}\\ &= \Vert{\boldsymbol{v}}\Vert\Vert{\boldsymbol{w}}\Vert \cos{(90)}\\ &= 0 \end{align*}$$

This completes the proof.

Theorem.

Commutative property of dot product

The commutative property of the dot product states that the ordering of the dot product between two vectors does not matter:

$$\boldsymbol{v}\cdot\boldsymbol{w}= \boldsymbol{w}\cdot\boldsymbol{v}$$

Proof. We will prove the case for $\mathbb{R}^3$ here, but the proof can easily be generalized for $\mathbb{R}^n$. Let vectors $\boldsymbol{v}$ and $\boldsymbol{w}$ be:

$$\boldsymbol{v}= \begin{pmatrix} v_1\\v_2\\v_3\\ \end{pmatrix},\;\;\;\; \boldsymbol{w}= \begin{pmatrix} w_1\\w_2\\w_3\\ \end{pmatrix}$$

Starting from the left-hand side:

$$\begin{align*} \boldsymbol{v}\cdot\boldsymbol{w}&= v_1w_1+v_2w_2+v_3w_3\\ &=w_1v_1+w_2v_2+w_3v_3\\ &=\boldsymbol{w}\cdot\boldsymbol{v} \end{align*}$$

This completes the proof.

Theorem.

Position of scalars can be changed

If $\boldsymbol{a}$ and $\boldsymbol{b}$ are vectors in $\mathbb{R}^n$ and $\lambda$ is some constant scalar, then:

$$(\lambda{\boldsymbol{v}})\cdot{\boldsymbol{w}}= \boldsymbol{v}\cdot(\lambda{\boldsymbol{w}}) =\lambda(\boldsymbol{v}\cdot\boldsymbol{w})$$

Proof. We will prove the case for $\mathbb{R}^3$ here, but the proof can easily be generalized for $\mathbb{R}^n$. Let vectors $\boldsymbol{v}$ and $\boldsymbol{w}$ be:

$$\boldsymbol{v}= \begin{pmatrix} v_1\\v_2\\v_3\\ \end{pmatrix},\;\;\;\; \boldsymbol{w}= \begin{pmatrix} w_1\\w_2\\w_3\\ \end{pmatrix}$$

Starting from the left-hand side:

$$\begin{align*} (\lambda\boldsymbol{v})\cdot{\boldsymbol{w}}&= \begin{pmatrix} \lambda{v_1}\\\lambda{v_2}\\\lambda{v_3}\\ \end{pmatrix}\cdot \begin{pmatrix} w_1\\w_2\\w_3\\ \end{pmatrix}\\ &=\lambda{v_1w_1}+\lambda{v_2w_2}+\lambda{v_3w_3} \end{align*}$$

To prove the first equality:

$$\begin{align*} (\lambda\boldsymbol{v})\cdot{\boldsymbol{w}} &=\lambda{v_1w_1}+\lambda{v_2w_2}+\lambda{v_3w_3}\\ &=v_1(\lambda{w_1})+v_2(\lambda{w_2})+v_3(\lambda{w_3})\\ &=\boldsymbol{v}\cdot(\lambda{\boldsymbol{w}}) \end{align*}$$

To prove the second equality:

$$\begin{align*} (\lambda\boldsymbol{v})\cdot{\boldsymbol{w}} &=\lambda{v_1w_1}+\lambda{v_2w_2}+\lambda{v_3w_3}\\ &=\lambda(v_1w_1)+\lambda(v_2w_2)+\lambda(v_3w_3)\\ &=\lambda(\boldsymbol{v}\cdot\boldsymbol{w}) \end{align*}$$

This completes the proof.

Theorem.

Dot Product Distributes Over Vector Addition

If $\boldsymbol{u}$, $\boldsymbol{v}$ and $\boldsymbol{w}$ are vectors in $\mathbb{R}^n$, then:

$$\begin{align*} (\boldsymbol{u}+\boldsymbol{v})\cdot\boldsymbol{w}= \boldsymbol{u}\cdot\boldsymbol{w}+\boldsymbol{v}\cdot\boldsymbol{w} \end{align*}$$

Proof. We will prove the case for $\mathbb{R}^3$ here, but the proof can easily be generalized for $\mathbb{R}^n$. Let vectors $\boldsymbol{u}$, $\boldsymbol{v}$ and $\boldsymbol{w}$ be as follows:

$$\boldsymbol{u}= \begin{pmatrix} u_1\\u_2\\u_3\\ \end{pmatrix},\;\;\;\; \boldsymbol{v}= \begin{pmatrix} v_1\\v_2\\v_3\\ \end{pmatrix},\;\;\;\; \boldsymbol{w}= \begin{pmatrix} w_1\\w_2\\w_3\\ \end{pmatrix}$$

The vector $\boldsymbol{u}+\boldsymbol{v}$ is:

$$\boldsymbol{u}+\boldsymbol{v}= \begin{pmatrix} u_1+v_1\\u_2+v_2\\u_3+v_3\\ \end{pmatrix}$$

The left-hand side $(\boldsymbol{u}+\boldsymbol{v})\cdot\boldsymbol{w}$ is therefore:

$$\begin{align*} (\boldsymbol{u}+\boldsymbol{v})\cdot\boldsymbol{w}&= \begin{pmatrix} u_1+v_1\\u_2+v_2\\u_3+v_3\\ \end{pmatrix}\cdot \begin{pmatrix} w_1\\w_2\\w_3 \end{pmatrix}\\&= w_1u_1+w_1v_1+ w_2u_2+w_2v_2+ w_3u_3+w_3v_3 \\&=(w_1u_1+w_2u_2+w_3u_3)+(w_1v_1+w_2v_2+w_3v_3) \\&=\boldsymbol{u}\cdot{\boldsymbol{w}}+\boldsymbol{v}\cdot\boldsymbol{w} \end{align*}$$

This completes the proof.

Relationship between magnitude and dot product

The following relationships between vector magnitude and dot product is useful for future proofs.

Theorem.

Dot product of a vector with itself is equal to the square of the vector's magnitude

The magnitude of a vector $\boldsymbol{v}$ can be expressed as dot product like so:

$$\Vert{\boldsymbol{v}}\Vert =(\boldsymbol{v}\cdot\boldsymbol{v})^{1/2}$$

Taking the square on both sides gives:

$$\Vert{\boldsymbol{v}}\Vert^2 =\boldsymbol{v}\cdot\boldsymbol{v}$$

Proof. Consider the following vector:

$$\boldsymbol{v}=\begin{pmatrix} v_1\\ v_2\\ \vdots\\ v_n\\ \end{pmatrix}$$

By definition, the magnitude of $\boldsymbol{v}$ is:

$$\begin{align*} \Vert\boldsymbol{v}\Vert &=(v_1^2+v_2^2+\cdots+v_n^2)^{1/2}\\ &=\Big((v_1)(v_1)+(v_2)(v_2)+\cdots+(v_n)(v_n)\Big)^{1/2}\\ &=(\boldsymbol{v}\cdot\boldsymbol{v})^{(1/2)} \end{align*}$$

This completes the proof.

Theorem.

Expressing dot product of two vectors using magnitude (2)

If $\boldsymbol{v}$ and $\boldsymbol{w}$ are vectors, then:

$$\boldsymbol{v}\cdot{\boldsymbol{w}}= \frac{1}{4}\Vert\boldsymbol{v}+\boldsymbol{w}\Vert^2- \frac{1}{4}\Vert\boldsymbol{v}-\boldsymbol{w}\Vert^2$$

Proof. Let's start from the right-hand side:

$$\begin{align*} \Vert\boldsymbol{v}+\boldsymbol{w}\Vert^2&= (\boldsymbol{v}+\boldsymbol{w})\cdot(\boldsymbol{v}+\boldsymbol{w})\\ &= (\boldsymbol{v}\cdot\boldsymbol{v})+ 2(\boldsymbol{v}\cdot\boldsymbol{w})+(\boldsymbol{w}\cdot\boldsymbol{w})\\ &= \Vert\boldsymbol{v}\Vert^2+ 2(\boldsymbol{v}\cdot\boldsymbol{w})+ \Vert\boldsymbol{w}\Vert^2\\ \end{align*}$$

Note that the first step follows from theoremlink.

Similarly, we have that:

$$\begin{align*} \Vert\boldsymbol{v}-\boldsymbol{w}\Vert^2&= (\boldsymbol{v}-\boldsymbol{w})\cdot(\boldsymbol{v}-\boldsymbol{w})\\ &= (\boldsymbol{v}\cdot\boldsymbol{v})- 2(\boldsymbol{v}\cdot\boldsymbol{w})+(\boldsymbol{w}\cdot\boldsymbol{w})\\ &= \Vert\boldsymbol{v}\Vert^2- 2(\boldsymbol{v}\cdot\boldsymbol{w})+ \Vert\boldsymbol{w}\Vert^2\\ \end{align*}$$

Therefore:

$$\begin{align*} \frac{1}{4}\Vert\boldsymbol{v}+\boldsymbol{w}\Vert^2- \frac{1}{4}\Vert\boldsymbol{v}-\boldsymbol{w}\Vert^2&= \frac{1}{4}( \Vert\boldsymbol{v}\Vert^2+ 2(\boldsymbol{v}\cdot\boldsymbol{w})+ \Vert\boldsymbol{w}\Vert^2 )-\frac{1}{4} (\Vert\boldsymbol{v}\Vert^2- 2(\boldsymbol{v}\cdot\boldsymbol{w})+ \Vert\boldsymbol{w}\Vert^2)\\ &=\frac{1}{2}(\boldsymbol{v}\cdot\boldsymbol{w})+ \frac{1}{2}(\boldsymbol{v}\cdot\boldsymbol{w})\\ &=\boldsymbol{v}\cdot\boldsymbol{w} \end{align*}$$

This completes the proof.

Practice problems

Suppose we have the following vectors:

$$\boldsymbol{v}= \begin{pmatrix} 3\\2 \end{pmatrix},\;\;\;\; \boldsymbol{w}= \begin{pmatrix} 1\\4 \end{pmatrix}$$

Compute their dot product.

Your answer
Submit answer
Show solution

The product of the two vectors is:

$$\begin{align*} \boldsymbol{v}\cdot \boldsymbol{w}&= \begin{pmatrix}3\\2\end{pmatrix}\cdot \begin{pmatrix}1\\4\end{pmatrix}\\ &=(3)(1)+(2)(4)\\ &=11\\ \end{align*}$$

Consider the following two vectors:

$$\boldsymbol{v}= \begin{pmatrix} 1\\3 \end{pmatrix},\;\;\;\; \boldsymbol{w}= \begin{pmatrix} 3\\4 \end{pmatrix}$$

Find the angle (rounded to 1st decimal place) between the two vectors.

Your answer
Submit answer
Show solution

We use theoremlink to compute the angle between two vectors:

$$\cos{(\theta)}= \frac{\boldsymbol{v}\cdot{\boldsymbol{w}}} {\Vert{\boldsymbol{v}}\Vert\Vert{\boldsymbol{w}}\Vert}$$

Where $\Vert{\boldsymbol{v}}\Vert$ and $\Vert{\boldsymbol{w}}\Vert$ are the magnitude of $\boldsymbol{v}$ and $\boldsymbol{w}$ respectively, while $\theta$ is the angle we are after.

The numerator is:

$$\begin{align*} \boldsymbol{v}\cdot\boldsymbol{w} &=(1)(3)+(3)(4)\\ &=15 \end{align*}$$

The denominator is:

$$\begin{align*} \Vert\boldsymbol{v}\Vert\Vert\boldsymbol{w}\Vert&= \left(\sqrt{1^2+3^2}\right) \left(\sqrt{3^2+4^2}\right)\\ &=5\sqrt{10} \end{align*}$$

The angle $\theta$ is:

$$\cos(\theta)=\frac{15}{5\sqrt{10}}\approx18.4^\circ$$

Consider the following vectors:

$$\boldsymbol{v}= \begin{pmatrix} 4\\2 \end{pmatrix},\;\;\;\; \boldsymbol{w}= \begin{pmatrix} -1\\2 \end{pmatrix}$$

Show that the two vectors are perpendicular.

Show solution

To show that these vectors are perpendicular, we can use theoremlink and compute their dot product:

$$\begin{align*} \boldsymbol{v}\cdot\boldsymbol{w}&= (4)(-1)+(2)(2)\\&= 0 \end{align*}$$

Since their dot product is zero, $\boldsymbol{v}$ and $\boldsymbol{w}$ must be perpendicular.

Consider the following vector:

$$\boldsymbol{v}=\begin{pmatrix}5\\3\end{pmatrix}$$

Use dot product to compute the square of its magnitude, that is, $\Vert{\boldsymbol{v}}\Vert^2$.

Your answer
Submit answer
Show solution

By theoremlink, we know that the squared magnitude is equal to the dot product of the vector with itself. This means that:

$$\begin{align*} \Vert{\boldsymbol{v}}\Vert^2&= \boldsymbol{v}\cdot\boldsymbol{v}\\ &=5^2+3^2\\ &=34 \end{align*}$$
robocat
Published by Isshin Inada
Edited by 0 others
Did you find this page useful?
thumb_up
thumb_down
Comment
Citation
Ask a question or leave a feedback...